Modelo de Bohr del átomo de hidrógeno - Niveles de energía del átomo de hidrógeno

Por qué las órbitas permitidas (estacionarias) corresponden a aquellas para las cuales el momento angular orbital del electrón es un múltiplo entero de = h 2 π ?

L = norte
Regla de Cuantización de Bohr del momento angular que conduce a los niveles de energía del átomo de Hidrógeno:
mi norte = Z 2 1 + metro mi METRO pag norte 2

la pregunta es ¿Por qué los números cuánticos son números naturales ?

mi = norte h v
norte = 0 , 1 , 2 , 3....

Porque las constantes fueron escogidas de esa manera.
Esto se explica en detalle en la página de Wikipedia sobre el modelo de Bohr y el "principio de correspondencia", ¿qué no está claro?
Es posible que desee ver también en.wikipedia.org/wiki/De_Broglie_hypothesis .
Los fotones no pueden estar en partes fragmentadas, por lo que n no puede ser decimal. Por lo tanto, n es siempre un número entero.

Respuestas (2)

El modelo de Bohr no era la teoría correcta de toda la física atómica pero describía correctamente los niveles del átomo de Hidrógeno, debido a una coincidencia matemática relacionada con este problema matemático solucionable en toda la mecánica cuántica.

Las condiciones de integralidad del modelo de Bohr se eligieron ad hoc, de modo que los niveles de energía que se ven en los espectros de absorción/emisión de hidrógeno pudieran coincidir, pero el punto de partida más justificado para derivarlos fue la condición de cuantificación de Sommerfeld-Wilson.

0 T pag r d q r = norte h
donde la integral de pag d q recorre un período orbital. De esta forma, es análogo a la declaración en la mecánica cuántica completa, que reemplazó el modelo de Bohr a mediados de 1920, que el espacio de fase (espacio parametrizado por las posiciones q y momentos pag ) se compone de celdas cuya área (o volumen) es igual a múltiplos de h = 2 π (o poderes de h , si hay muchas coordenadas). La órbita rodea un área en el espacio de fase y el área debe cuantificarse.

Por coincidencia, esto también es más o menos equivalente a la cuantización del momento angular, L = norte . En la mecánica cuántica, se cumplen condiciones similares pero por razones ligeramente diferentes y la cuantización del momento angular también permite valores semiintegrales: j = norte / 2 dónde norte es entero según la mecánica cuántica.

Hay que separar las explicaciones del modelo de Bohr de las de la mecánica cuántica propiamente dicha; no son equivalentes porque los modelos tampoco son equivalentes. Y no tiene mucho sentido pensar en el origen de las condiciones en el modelo de Bohr porque el modelo de Bohr fundamentalmente no es la teoría correcta tal como la conocemos hoy.

En la mecánica cuántica completa, uno puede encontrar varios hechos de "cuantificación" con el cuántico proporcional a h o o / 2 . Todos ellos tienen un origen cuántico pero la explicación detallada es diferente para cada uno: la cuantización del momento angular; la celda elemental del espacio de fase; los cambios no físicos de la acción por un múltiplo de h .

Si bien el modelo de Bohr no es correcto, la justificación del modelo de Bohr por el principio de correspondencia también es fundamentalmente correcta en la comprensión moderna --- la razón de la regla de cuantización es que el fotón emitido debe tener el período clásico en los movimientos clásicos.
Estimado Ron, no estoy seguro en qué sentido puedes tener explicaciones "fundamentalmente correctas" usando teorías que no son correctas. Algo sobre pag d q se cuantifica en unidades de h en la teoría correcta, pero lo que esta cantidad cuantizada claramente no es exactamente la integral durante un período en un modelo de átomo de Bohr incorrecto, por lo que el acuerdo es vago y una coincidencia.
No es una coincidencia. El modelo de Bohr es una imagen equivocada, pero la idea de que el nivel de separación cerca de la energía E debe ser igual al período clásico de la órbita con energía E es correcta en números cuánticos grandes, y debe serlo, porque el argumento de correspondencia de Bohr es básicamente sólido. Si acopla un oscilador clásico al campo EM débilmente, las ondas EM emitidas tienen una frecuencia igual a la frecuencia orbital clásica. ω (y múltiplos enteros de los mismos), por lo que si se emiten fotones, los niveles de energía del oscilador deben estar espaciados con espaciado h ω .
El punto clave es: espaciamiento de niveles de energía = frecuencia clásica = (2pi/período clásico). Esta es la identidad que usa Bohr para encontrar todo lo demás. Debe ser así para que el acoplamiento de fotones tenga sentido, y es la razón por la que la teoría de Bohr fue más que una conjetura, sino una semilla adecuada para que Heisenberg completara la mecánica de matrices.
Querido Ron, todas estas cosas funcionan para el átomo de hidrógeno, un solo problema y un solo conjunto de niveles con mi 1 / norte 2 , y no sirven para nada más e incluso en este caso, su descripción conduce a una interpretación incorrecta de las energías, etc. Por eso el acuerdo es una coincidencia.
Estas cosas funcionan para todos los sistemas cuánticos con un límite clásico integrable, el espaciado de niveles siempre es asintótico a 2-pi durante el período clásico (si hay muchos períodos, como en un sistema multiperiódico, tiene un nivel de energía multiperiódico patrón, con espaciamientos independientes, pero si el período es infinito, se rompe). Esta es la validez asintótica conocida de la aproximación WKB, que es equivalente a la condición de Bohr, y el argumento de Bohr es exactamente correcto. No depende de un problema especial. Lea el argumento de Bohr antes de comentar, ya lo he dado tres veces aquí.
El modelo de Bohr no era la teoría correcta acordada
@BadBoy: el modelo de Bohr no era la teoría correcta, pero era la teoría semiclásica correcta . No es arbitrario, y el argumento de Bohr es correcto. Las órbitas de Bohr deben tener un espaciado de energía igual a 2pi durante el período clásico, y existe una correspondencia entre los componentes de Fourier de un movimiento clásico y los elementos de posición de la matriz fuera de la diagonal, esto es lo que descubrió Bohr y condujo a la mecánica matricial. La gente ignora esto, porque es un argumento diferente al que aparece en los libros. Pero es 100% correcto para el orden semiclásico, es una calumnia para Bohr decir lo contrario.
Estimado Ron, soy un gran admirador de Bohr, pero es absurdo decir que el modelo de Bohr era un modelo correcto de cualquier cosa cuando se construyó décadas antes de que la gente supiera que todas las leyes de la física en realidad solo predecían probabilidades, etc. No había una teoría probabilística de las energías antes de la década de 1920 y la teoría correcta en cualquier límite o fuera de cualquier límite tiene que ser probabilística. De ello se deduce que la teoría de Bohr no era la teoría correcta. En mi respuesta, de hecho elegí pag d q ya que es el más cercano a QM pero no tenía QM en 1910, ni siquiera en un límite.
@LubošMotl: Sí, es cierto, nadie tenía una idea clara de las transiciones de nivel estadístico hasta los coeficientes A,B de Einstein, pero la teoría de Bohr reproduce los niveles de energía semiclásicos correctos (en el límite N grande) para cualquier sistema integrable y esto no es un coincidencia. La razón es que existe una condición de consistencia: el nivel de separación debe ser tal que la emisión de fotones debe ser posible en las frecuencias clásicas (múltiplos enteros de la frecuencia orbital). Este es el principio de correspondencia de Bohr, y realmente es correcto que los niveles de energía sigan el orden principal en . Lee mi respuesta.

La regla básica que justificó la cuantización del momento angular se puede derivar de la siguiente manera, a partir del principio de correspondencia. Esta derivación es heurística e inexacta, y solo se vuelve absolutamente correcta una vez que conoce la mecánica cuántica completa.

Considere un electrón que orbita alrededor de un núcleo de carga 1 (protón, deuterón o tritón) en un radio grande R con un momento angular L. El período clásico se encuentra al igualar la fuerza centrípeta a la atracción electrostática del electrón al núcleo:

metro v 2 R = mi 2 R 2

Dónde 1 4 π ϵ 0 factor se absorbe en la constante e. Encuentras la velocidad orbital v a partir de esto, así que

v = mi 2 metro R

Que te dice cuánto tiempo dar la vuelta al círculo T = 2 π R v . Entonces la frecuencia angular ω de la órbita

2 π T = ω = mi 2 metro R 3

La energía cinética del electrón en órbita se encuentra directamente a partir de la fórmula centrípeta:

metro v 2 2 = mi 2 2 R

La energía potencial es:

mi 2 R

Entonces la energía total es la mitad de la energía potencial negativa

mi 2 2 R

Clásicamente, este sistema irradiará ondas electromagnéticas que son periódicas con período T. Esto significa que la radiación saliente tiene una frecuencia ω . Cuánticamente, el electrón en órbita solo puede emitir fotones con grumos discretos de energía, y esto significa que la energía solo puede cambiar en pasos de ω , que es la energía de un fotón de frecuencia ω .

Esto significa que si tiene una emisión de fotones consistente, las energías deben estar espaciadas en niveles de energía discretos, y el espacio entre dos niveles adyacentes en R grande es igual a la frecuencia orbital clásica:

Δ mi = ω = mi 2 metro R 3

Esta condición significa que si hay un nivel de energía en mi , hay otro nivel de energía en mi Δ mi (donde terminas después de una emisión de fotones), luego otra en mi 2 Δ mi en pasos discretos.

Todo esto es un razonamiento semiclásico, y solo funciona realmente cuando el espaciado Δ mi es mucho menor que la energía cinética y la energía potencial. El espaciado llega a cero como la potencia 3/2 del radio, por lo que esta aproximación es válida para órbitas grandes.

También puedes calcular el espacio R entre órbitas adyacentes

mi = mi 2 2 R
d mi = mi 2 2 R 2 d R

Entonces, el espaciado en E se traduce en un espaciado en R (en la aproximación de que Δ mi y por lo tanto Δ R son pequeños para que se aproximen a los diferenciales infinitesimales anteriores)

Δ R = 2 R 2 mi 2 Δ mi = 2 R metro mi 2

El cambio de E y R en cada paso es complicado, pero tiene las mismas unidades que el momento angular, y puede calcular el cambio en el momento angular cuando realiza un solo paso:

L = metro v R = mi 2 metro R
d L = 1 2 mi 2 metro R d R

De modo que

Δ L = 1 2 mi 2 metro R Δ R =

Esto es muy simple --- el momento angular está espaciado en múltiplos enteros de en grandes órbitas circulares. A partir de esto, uno puede hacer la conjetura plausible de que esto es cierto para todos los números cuánticos, grandes y pequeños, y luego sigue el modelo de Bohr.

La generalización de este argumento para derivar la antigua condición cuántica es considerar el período de las órbitas clásicas y hacer que el espaciado de energía sea igual a veces la frecuencia orbital de un sistema general. Este requisito significa que semiclásicamente:

j = pag d q = 2 π norte = norte h

Esto se muestra en la página de Wikipedia para el principio de correspondencia . La misma cantidad J es un invariante adiabático , no cambia bajo deformaciones lentas de un sistema clásico, y la cantidad cuantificada debe tener esta propiedad, ya que una deformación lenta no tiene las altas frecuencias requeridas para hacer transiciones de estado en mecánica cuántica . Este argumento se resume en la página de wikipedia sobre los invariantes adiabáticos .